LSAT and Law School Admissions Forum

Get expert LSAT preparation and law school admissions advice from PowerScore Test Preparation.

 Administrator
PowerScore Staff
  • PowerScore Staff
  • Posts: 8916
  • Joined: Feb 02, 2011
|
#25110
Complete Question Explanation
(The complete setup for this game can be found here: lsat/viewtopic.php?t=4009)

The correct answer choice is (D)

G can only be placed on the top shelf in Templates #2 and #3.

Answer choices (A) and (B) both contain just two books, indicating that the 2-2-4 distribution is in play, and thus that Template #3 applies.

In Template #3, the removal of G from the bottom shelf forces H and L to be placed on the bottom shelf. With G on the top shelf, O and K then rotate between the top shelf and middle shelf:
powerscore_M12_T4_O2011_LG_explanations_game_4_#22_diagram_1.png
Thus, answer choices (A) and (B) are both incorrect because neither contains K or O.

Answer choices (C), (D), and (E) each contain three books, indicating that Template #2 is in effect. In this template, when G is placed on the top shelf, O, K, and the remainder of H/L fill the final three places:
powerscore_M12_T4_O2011_LG_explanations_game_4_#22_diagram_2.png
Answer choice (C) is incorrect because it contains both H and L, which is impossible because of one of the two must be placed on the bottom shelf.

Answer choice (D) is possible, and is the correct answer.

Answer choice (E) cannot occur because both F and M must be on the bottom shelf in Template #2.
You do not have the required permissions to view the files attached to this post.
 jared.xu
  • Posts: 65
  • Joined: Oct 07, 2011
|
#3180
G seems to be a random in this game. And so this local question that places G in the top shelf does not allow me to make any inferences. I ended up having to try each answer choices out and wasting a lot of time. Please tell me how you would attack this question, and save time. Thank you.
 Nikki Siclunov
PowerScore Staff
  • PowerScore Staff
  • Posts: 1362
  • Joined: Aug 02, 2011
|
#3195
The first rule of the game creates two distributions of books to shelves: 2-3-3 or 2-2-4. The FM block must be lower than K, whereas L must be lower than O.

If G is placed on the top shelf, make two local diagrams to represent each distributional possibility:

Distribution 2-2-4

In this distribution, FM must be on the bottom shelf since I is in the middle. However, you need to put 4 books on the bottom shelf, and neither K nor O can go there. Figure out what's left to be on the bottom, and you can quickly eliminate the first two answer choices.

Distribution 2-3-3

In this distribution, FM must also be on the bottom shelf, because if it were in the middle shelf, you'd have to put O and K on the top , along with G - which is impossible. This eliminates (E).

Given that the bottom shelf has 3 books in the second distribution, two of which are F and M, proceed just like we did before - figure out what's left to occupy the third shelf. Even though we don't know if the book will be H or L, they cannot both be in the middle - this eliminates (C). Hurdle the Uncertainty :-)

Get the most out of your LSAT Prep Plus subscription.

Analyze and track your performance with our Testing and Analytics Package.